LSAT and Law School Admissions Forum

Get expert LSAT preparation and law school admissions advice from PowerScore Test Preparation.

 Administrator
PowerScore Staff
  • PowerScore Staff
  • Posts: 8917
  • Joined: Feb 02, 2011
|
#35208
Complete Question Explanation
(See the complete passage discussion here: lsat/viewtopic.php?t=14217)

The correct answer choice is (E)

This question asks for the choice that best describes the overall organization of the passage. As
discussed in the VIEWSTAMP analysis above, the author introduces an issue, presents a study that
deals with the issue, and discusses the valuable findings of the study.

Answer choice (A): The author does not judge the results of the study to be inconclusive, so this
choice fails the Fact Test and cannot be the right answer to this Must Be True question.

Answer choice (B): The author does not support, then undermine, then modify a hypothesis as
described in this answer, so this choice should be ruled out of contention.

Answer choice (C): While a study is evaluated, and a course of action recommended, the likely
effectiveness of the plan is not discussed. This choice does not describe the passage’s organization.
Answer choice (D): Only one study is discussed in the passage, and there is no modification of the
goal’s objectives, nor is a methodology to achieve a revised goal detailed.

Answer choice (E): This is the correct answer choice; the author of the passage presents a
problem, describes a study, and bases a recommended course of action on the results of the study
 alee
  • Posts: 57
  • Joined: Mar 21, 2012
|
#5661
Hi guys,

In Q21 of PT 65 Section 3, how do you choose between the following 2 options:

The incorrect one I found tempting was:

(A) A study is described, the results of the study are scrutinized, and the results are judged to be inconclusive but promising.

I did eventually go with the right one, but really I could have gone with either:

(E) A problem is presented, a study addressing the problem is described, and a course of action based on the study's findings is given.


What's wrong with A exactly? I guess that its never explicitly stated that the results are inconclusive, but it *does* say that the final potential course of action is just based on what 'Researchers now believe', and that perhaps warrants the descriptor of 'inconclusive'.

Perhaps a second point is that the results are not scrutinized (i.e. *critically* discussed), they are just presented without critical discussion of them.

1. Is that (reasons above) what knocks out A?

2. Regarding why E is right, I am a little convinced that a 'course of action is given' based on the study's findings. Perhaps the reason why I am unconvinced is that I read given as 'recommended', and whilst a action based on the study is *described* it's never explicitly recommended by the author, it's just presented as something that 'researchers now believe'.

Whew, long post, thanks for reading!
 Adam Tyson
PowerScore Staff
  • PowerScore Staff
  • Posts: 5153
  • Joined: Apr 14, 2011
|
#5877
Hey alee,

Again, good analysis on this passage and the answers. I agree that A should be rejected in part because there is no scrutiny of the results, just a presentation of them. Another reason why E is better is that it correctly starts off by including that "a problem is presented", while none of the other answers include that important element of the structure.

I would not say that "given" is the same as "recemmended". I would say that "given" does more nearly mean presented or stated in this instance. No reason to reject E on those grounds.

Good job - keep it up!

Adam
 deck1134
  • Posts: 160
  • Joined: Jun 11, 2018
|
#49270
Hi PowerScore,

I concluded that the "effectiveness" point was concluded in line 53 when they talk about "preliminary results suggest."

Isn't that indicating that it is likely to occur?

Thanks.
 Adam Tyson
PowerScore Staff
  • PowerScore Staff
  • Posts: 5153
  • Joined: Apr 14, 2011
|
#49332
I assume you are talking about answer C here, Deck? I wouldn't describe those lines as conclusions about the likely effectiveness, but only as describing something that would be required in order for the plan to be effective.

More importantly, though, this answer should have been discarded as a loser by the time you reached the first comma, because "a study is evaluated" doesn't happen until after the problem is described. That answer ignores the problem, which is the crucial starting point of the passage and an essential element of any description of the structure! Answer E is the only answer that starts at the beginning, and since structure answers need to describe everything important that was presented, in the order in which it was presented, all the other answers start out dead in the water.

Get the most out of your LSAT Prep Plus subscription.

Analyze and track your performance with our Testing and Analytics Package.